Use Pythagoras to find the height and hence, the area of the triangle
below. Give height to 1 decimal place and area to the nearest whole. Write
answer in format: h= A= *
20 mm
Val

Answers

Answer 1

Answer:

h=17.3 A=173

Step-by-step explanation:

Calculator

Answer 2

Answer:

height = 17.3 mm

area = 173 mm²

Step-by-step explanation:

all three sides are of the same length (20 mm).

so, the height actually splits the baseline in half

(2 × 10 mm) while hitting it at a 90 degree angle.

so, we use Pythagoras, where the full side opposite of this 90 degree angle is c (Hypotenuse), the height of the main triangle is one side, and half of the baseline is the other side.

c² = a² + b²

20² = 10² + height²

400 = 100 + height²

300 = height²

height = 17.3 mm

the area of the main triangle is baseline (20) times height divided by 2.

so,

At = 20×17.3/2 = 10×17.3 = 173 mm²


Related Questions

32 x square Y - 2 y cube

Answers

Answer:

207y

Step-by-step explanation:

what is the relation that represents the relation

Answers

Answer:

what can i help u with

Step-by-step explanation:

I really can't help u with that sorry i am bad at math

Write an equation of the graph (shown below) in slope intercept form.

Answers

This is not a valid question and, therefore, cannot be answered. (Reason: No graph is attached.) Try revising it or giving more information. Hope this helps! ❤️

If the length of the shorter arc AB is 22cm and C is the center of the circle then the circumference of the circle
is:

Answers

Answer:

22= 2(pi)(r)(45/360)

28.01

C=176

Step-by-step explanation:

Determine what type of model best fits the given situation: the temperature of a cup of coffee decreases by 5 F every 20 minutes.

A. liner
B. exponential
C. quadratic
D. none of these

Answers

Answer: T = -t / 4 + T0   where t is the temperature in minutes elapsed, T is the final temperature, and T0 is the initial temperature

Explanation: This is a linear equation in T and t

(-1 / 4 represents -5 deg / 20 min  = - 1 deg / 4min

express the ratio 7day to 6weeks as a decimal fraction

Answers

Answer:

6 weeks=6*7 days=42days

7/42 =1/6 =0.16667

OR

7 days=1 week

therefore 1/6=0.16667

Note that both must be in the same unit.

Determine, to one decimal place, the length, width & height of the rectangular prism that would have the greatest volume, with a surface area of 200 cm^2.

Answers

Answer:

The length = The width = The height  ≈ 5.8 cm

Step-by-step explanation:

The volume of a rectangular pyramid, V = l × w × h

The surface area of the pyramid = 2 × l × h + 2 × w × h + 2 × l × w = 200

∴  l × h + w × h + l × w = 200/2 = 100

We have that the maximum volume is given when the length, width, and height are equal and one length is not a fraction of the other. Therefore, we get;

At maximum volume, l = w = h

∴ l × h + w × h + l × w = 3·l² = 100

l² = 100/3

l = 10/√3

Therefore, the volume, v = l³ = (10/√3)³

The length = The width = The height = 10/√3 cm ≈ 5.8 cm

solve the quadratic equation
give your answer to 2 decimal places
: 3x^2+x-5=0

Answers

Given:

The quadratic equation is:

[tex]3x^2+x-5=0[/tex]

To find:

The solution for the given equation rounded to 2 decimal places.

Solution:

Quadratic formula: If a quadratic equation is [tex]ax^2+bx+c=0[/tex], then:

[tex]x=\dfrac{-b\pm \sqrt{b^2-4ac}}{2a}[/tex]

We have,

[tex]3x^2+x-5=0[/tex]

Here, [tex]a=3,b=1,c=-5[/tex]. Using the quadratic formula, we get

[tex]x=\dfrac{-1\pm \sqrt{1^2-4(3)(-5)}}{2(3)}[/tex]

[tex]x=\dfrac{-1\pm \sqrt{1+60}}{6}[/tex]

[tex]x=\dfrac{-1\pm \sqrt{61}}{6}[/tex]

[tex]x=\dfrac{-1\pm 7.81025}{6}[/tex]

Now,

[tex]x=\dfrac{-1+7.81025}{6}[/tex]

[tex]x=1.13504167[/tex]

[tex]x\approx 1.14[/tex]

And

[tex]x=\dfrac{-1-7.81025}{6}[/tex]

[tex]x=-1.468375[/tex]

[tex]x\approx -1.47[/tex]

Therefore, the required solutions are 1.14 and -1.47.

evelins room has an area of 45 squar feet. the length of her room is 5. what is the perimeter of her room?

Answers

Answer:

28 feet

Step-by-step explanation:

Since the area is 45 and length is 5.In order to find the width we divide the area by the side given

45 ÷ 5 = 9 is the width

Perimeter is the sum of all the side of a figure.

9 + 9 + 5 + 5

= 28

I hope this helps :)

for the function g(x)=3-8(1/4)^2-x

a) State the y-intercept

b) State the equation of the horizontal asymptote

c) State whether the function is increasing or decreasing.

d) State the domain and range
e) Sketch the graph

Could anyone help?

Answers

Using function concepts, it is found that:

a) The y-intercept is y = 2.5.b) The horizontal asymptote is x = 3.c) The function is decreasing.d) The domain is [tex](-\infty,\infty)[/tex] and the range is [tex](-\infty,3)[/tex].e) The graph is given at the end of the answer.

------------------------------------

The given function is:

[tex]g(x) = 3 - 8\left(\frac{1}{4}\right)^{2-x}[/tex]

------------------------------------

Question a:

The y-intercept is g(0), thus:

[tex]g(0) = 3 - 8\left(\frac{1}{4}\right)^{2-0} = 3 - 8\left(\frac{1}{4}\right)^{2} = 3 - \frac{8}{16} = 3 - 0.5 = 2.5[/tex]

The y-intercept is y = 2.5.

------------------------------------

Question b:

The horizontal asymptote is the limit of the function when x goes to infinity, if it exists.

[tex]\lim_{x \rightarrow -\infty} g(x) = \lim_{x \rightarrow -\infty} 3 - 8\left(\frac{1}{4}\right)^{2-x} = 3 - 8\left(\frac{1}{4}\right)^{2+\infty} = 3 - 8\left(\frac{1}{4}\right)^{\infty} = 3 - 8\frac{1^{\infty}}{4^{\infty}} = 3 -0 = 3[/tex]

--------------------------------------------------

[tex]\lim_{x \rightarrow \infty} g(x) = \lim_{x \rightarrow \infty} 3 - 8\left(\frac{1}{4}\right)^{2-x} = 3 - 8\left(\frac{1}{4}\right)^{2-\infty} = 3 - 8\left(\frac{1}{4}\right)^{-\infty} = 3 - 8\times 4^{\infty} = 3 - \infty = -\infty[/tex]

Thus, the horizontal asymptote is x = 3.

--------------------------------------------------

Question c:

The limit of x going to infinity of the function is negative infinity, which means that the function is decreasing.

--------------------------------------------------

Question d:

Exponential function has no restrictions in the domain, so it is all real values, that is [tex](-\infty,\infty)[/tex].From the limits in item c, the range is: [tex](-\infty,3)[/tex]

--------------------------------------------------

The sketching of the graph is given appended at the end of this answer.

A similar problem is given at https://brainly.com/question/16533631

Fastest answer will be declared the brainliest

Answers

Answer:

All whole numbers are rational numbers.

False

All integers are whole numbers.

True

There are integers that are not rational numbers.

True

There are whole numbers that are not integers.

True.

Apples are cut into 8 pieces to be shared among some children. Twenty-two bags of seven apples are used. How many pieces of apple are cut?

Answers

Answer: 1232 pieces

Work Shown:

1 bag = 7 apples

22 bags = 22*7 = 154 apples

So we have 154 apples to work with in total.

Each of those apples is cut into 8 pieces, giving us 8*154 = 1232 pieces

We can write it as one single calculation to say 22*7*8 = 1232

Find the area of this circle. Use 3 for .
A = r2
12 cm
[?] cm2

Answers

Answer:

452.16 cm²

Step-by-step explanation:

Given :-

Radius = 12cm .

To find :-

Area of circle .

Solution :-

As we know that ,

A = πr² A = 3.14 * (12 cm)² A = 3.14 * 144cm² A = 452.16 cm²

Step-by-step explanation:

[tex]area = \pi {r}^{2} \\ = \pi \times {12}^{2} \\ = \pi \times 144 \\ = 3.14 \times 144 \\ = 452.16 {cm}^{2} \\ thank \: you[/tex]

What is the value of the expression below when w=2 and x=2
9w - x

Answers

Here, all we need to do is subtitute w = 2 and x = 2 into the expression, by multiplying and adding. First, we will muliply 9w:

9w

9 · w =

9 · 2 =

18

Now, all we have to do is use our answer for 9w, and subtract x from it:

9w - x

18 - (2) =

18 - 2 =

16

So, our answer is 16.

Let me know if you have any other questions! :)

Solve for x: -5 < 8x + 11 < 19

Answers

Answer:

-2<x<1

Step-by-step explanation:

-5 < 8x + 11 < 19

Subtract 11 from all sides

-5-11 < 8x + 11-11 < 19-11

-16 < 8x<8

Divide by 8

-16/8 < 8x/8 <8/8

-2<x<1

Question 2 Evaluate the expression 2(x - 3) + 3y when x = 5 and y = 3. Mark the correct answer.
A. 13
B. 15
C. 16
D. 25​

Answers

Answer:

A. 13

Step-by-step explanation:

2(x - 3) + 3y

2(5 - 3) + 3×3

2× 2 + 3×3

4 + 9

13

Answer:

The correct answer is A

Step-by-step explanation:

2(x-3) + 3y so you replace them and get 2( 5 - 3) +3(3)

next,

you would solve it

2 x 5 and 2 x 3 you get 10-6 + 9, the 9 is from multiplying 3 by 3

finally you solve it

10-6 = 4 +9

= 13!

9/7 = please answer me​

Answers

Answer:

[tex] \frac{9}{7} = 1.2857 = 1 \frac{2}{7} [/tex]

Answer:

Step-by-step explanation:

[tex]\\\\\\\\\\\\\\\\\\\\\\\\\\\\\\\\\\\\\\\\\\\\\\\\\\\\\\\la\la\la\la\ddddddddddddddddddddddddddddddddcleverdddddd\ffffffffffffffffffffffffffffffffffffffff\pppppppppppppppppppppppppppppppppppp\ddddddddddddddddddd\displaystyle\ \Large \boldsymbol {\frac{9}{7}=1\frac{2}{7} \ \ or \ \ 1.(28571)}[/tex]

Complete the table, and then use the drawing tools to create the graph representing the relationship between the amount of plant food remaining, f(x), and the number of days that have passed, x.

Answers

The complete table of the function is:

[tex]\begin{array}{cccccccc}x & {0} & {1} & {2} & {3} & {4}& {5} & {6} \ \\ f(x) & {72} & {60} & {48} & {36} & {24}& {12} & {0} \ \end{array}[/tex]

The equation missing from the question is:

[tex]y = 72 - 12x[/tex]

To complete the table, we simply calculate the y value for each x value;

When [tex]x = 0[/tex],     [tex]y = 72 - 12 * 0 = 72[/tex]

When [tex]x = 1[/tex],     [tex]y = 72 - 12 * 1 = 60[/tex]

When [tex]x = 2[/tex],     [tex]y = 72 - 12 * 2 = 48[/tex]

When [tex]x = 3[/tex],     [tex]y = 72 - 12 * 3 = 36[/tex]

When [tex]x = 4[/tex],     [tex]y = 72 - 12 * 4 = 24[/tex]

When [tex]x = 5[/tex],     [tex]y = 72 - 12 * 5 = 12[/tex]

When [tex]x = 6[/tex],     [tex]y = 72 - 12 * 6 = 0[/tex]

So, the complete table is:

[tex]\begin{array}{cccccccc}x & {0} & {1} & {2} & {3} & {4}& {5} & {6} \ \\ f(x) & {72} & {60} & {48} & {36} & {24}& {12} & {0} \ \end{array}[/tex]

To create a graph of f(x), we simply type [tex]y = 72 - 12x[/tex] on a drawing tool and the graph will be generated.

See attachment for graph

Learn more about graphs at:

https://brainly.com/question/20106471

Answer:

Step-by-step explanation:


Which number is a solution of the inequality?
3x - 15 ≥ 3

a.) -9/11

b.)6/11

c.)6

d.)5

Answers

Answer:

6

Step-by-step explanation:

[tex]3x - 15 \geqslant 3[/tex]

[tex]3x \geqslant 3 + 15[/tex]

[tex]x \geqslant \frac{18}{3} [/tex]

[tex]x \geqslant 6[/tex]

PLEASE HELP ASAP!!!!! The following graph shows a proportional relationship. What is the constant of proportionality between y and x in the graph?

Answers

Answer:

2/3

Step-by-step explanation:

The graph point is on (3,2)

Find the measure of the indicated angle to the nearest degree.

Answers

Answer:

Step-by-step explanation:

Adjacent to the undetermined angle is 6, and the hypotenuse has been given. We can conclude, after looking at our SOHCAHTOA, that we will be using cosine(CAH) to solve this problem.

Let the unspecified angle be [tex]\theta\\[/tex] (This sign is called theta, which is just a sign for angle)

Lets start!

cos[tex]\theta\\[/tex] = adj/hyp

cos[tex]\theta\\[/tex] = 6/13

[tex]\theta\\[/tex] = [tex]cos^{-1}[/tex](6/13)

[tex]\theta\\[/tex] = 62.5

[tex]\theta\\[/tex] = 63

Hope that helped!

Write the equation of the line that passes through the points (−7,5) and (-7,-8).
Put your answer in fully reduced point-slope form, unless it is a vertical or horizontal line.

Answers

Answer:

x = -7

Step-by-step explanation:

First we find the slope using

m = ( y2-y1)/(x2-x1)

   = ( -8 - 5)/( -7 - -7)

   = (-8-5)/(-7+7)

   = -13/0

This means the slope is undefined and the line is vertical

Vertical lines are in the form

x= constant and the constant is the x value of the points

x = -7

If P =
1 2
then, prove that P2 - 2P - 5 = 0, 1 = 0, where I and 0 are unit
3 1
matrix and null matrix of order 2 x 2 respectively.

Answers

Answer:

thanks for free ponits you dint gave it free but I theft I'm sorry

A P E X fasted answer is getting 30 points please hurry.

suppose a normal distribution has a mean of 98 and a standard deviation of 6. what is p(x ≥ 92)

Answers

Answer:

Step-by-step explanation:

If the mean is 98 and standard deviation is 6, then 92 is one standard deviation to the left of the mean; that means that 34% + 34% + 13.5% + 2.35% + .15% of the data is included in this area. Add those up and you get, as a percentage, 84%.

Answer:

0.84

Step-by-step explanation:

yee its right

Which shapes have the greatest area?
(Select all that apply.)

Answers

Answer:

the trapiezium

Step-by-step explanation:

It has a area of 4 which is the highest

Darryl has written 60 percent, or 12 pages, of his history report. Darryl wants to figure out how many total pages he needs to write. Darryl’s work is shown below.
Step 1: Write 60 percent as a ratio. StartFraction part Over whole EndFraction = StartFraction 60 Over 100 EndFraction

Answers

Answer:

total pages = 20

Step-by-step explanation:

60% of an unknown number is 12

Let the unknown number (total pages) be x.

60/100 of x = 12

60/100 * x = 12

3/5 x = 12

x = 12 * 5/3

x = 20

Total page equals 20 your welcome my friend

find the measure of one exterior angle for the following regular polygon ​

Answers

Answer:

36 degrees

Step-by-step explanation:

10 corners/sides.

the sum of all exterior angles in a polygon is always 360 degrees.

so, one exterior angle here is 360/10 = 36 degrees

PLS HELP WILL MAKE FIRST RIGHT ANSWER GETS BRAINLIEST ​

Answers

Answer:
26. A
27. A

Explanation:

(x^2+1)(x-1)=0 help me pls

Answers

Answer:

x = ±i ,  x=1

Step-by-step explanation:

(x^2+1)(x-1)=0

Using the zero product property

x^2 +1 = 0   x-1= 0

x^2 = -1       x=1

Taking the square root of the equation on the left

sqrt(x^2) = sqrt(-1)

x = ±i   where  i is the imaginary number

We still have x=1 from the equation on the right

If the lengths of the legs of a right triangle are 4 and 8, what is the length of the hypotenuse?

PLEASE HELP

Answers

Answer:

[tex]4\sqrt{5}[/tex]

Step-by-step explanation:

In order to solve this problem, we can use the pythagorean theorem, which is

a^2 + b^2 = c^2, where and b are the legs of a right triangle and c is the hypotenuse. Since we are given the leg lengths, we can substitute them in. So, where a is we can put in a 4 and where b is we can put in an 8:

a^2 + b^2 = c^2

(4)^2 + (8)^2 = c^2

Now, we can simplify and solve for c:

16 + 64 = c^2

80 = c^2

c = [tex]\sqrt{80}[/tex]

Our answer is not in simplified radical form because the number under is divisible by a perfect square, 16. We can divide the inside, 80,  by 16, and add a 4 on the outside, as it is the square root of 16:

c = [tex]4\sqrt{5}[/tex]

The length of the hypotenuse in the given right triangle, with legs measuring 4 and 8, is approximately 8.94.

To find the length of the hypotenuse in a right triangle, we can use the Pythagorean theorem. According to the theorem, in a right triangle, the square of the hypotenuse's length is equal to the sum of the squares of the lengths of the other two sides.

In this case, let's label the lengths of the legs as 'a' and 'b', with 'a' being 4 and 'b' being 8. The hypotenuse, which we need to find, can be represented as 'c'.

Applying the Pythagorean theorem, we have:

[tex]a^2 + b^2 = c^2[/tex]

Substituting the given values:

[tex]4^2 + 8^2 = c^2[/tex]

16 + 64 = [tex]c^2[/tex]

80 = [tex]c^2[/tex]

To find the length of the hypotenuse 'c', we need to take the square root of both sides:

√80 = √ [tex]c^2[/tex]

√80 = c

The square root of 80 is approximately 8.94.

Therefore, the length of the hypotenuse in the given right triangle, with legs measuring 4 and 8, is approximately 8.94.

To know more about  hypotenuse , here

https://brainly.com/question/2217700

#SPJ2

Other Questions
Write the sentences correctly pls 1 All his friends will be there except Peter.This button look as if it will match the material. They divided the chips equally among the three children.Where did the Egyptian people live at?The football player practiced for an hour write a letter to reply to a brother or sister who need money to buy book About a Month And A Half I Got A 10x & 25x Aquarium I Had 9 Fish And Betta Pebbles And In Less Than a Week 1 to 2 Fish Die A Day And I Believe Its The Betta Rocks Because The Rock Package Says Medicine For Bettas But Lethal To Fish, Bc There's Medicine In The Rocks That Affect The Water And It Helps Betta Fish But Lethal To Other Fishes.I Have 2 Turtles And Almost a Month There Living In Betta Rocks And There OK. which best describes a bacterium Which of the following is an opinion adverb Our guests can go to the pool during the dayThey can also go to the gym during the dayFinally, they can go to the hotel restaurant at nightWhich sentence best restates this information?A) The pool and gym are for daytime hotel guests, and the restaurant is for night time hotel guests.B) The gym, pool, and restaurant are options for our hotel guests.C) Guests can go to the gym or the pool during the day and the restaurant at night.D) Guests can go to the pool or gym before going to the restaurant at night.E) Guests can go to the gym or the pool before they go to the restaurant at night indentify the explicit function for the sequence in the table Im new to this app and I need help with those two questions please help!! Find the area of each figure. Round to the nearest tenth if necessary. Grammar Exercises ). Choose the correct answer from a, b, cord: * My daughter.....her teacher had given her a reward that day. a) agreed b) told c) said d) complained What heritages of your community make you feel proud ? Write a paragraph Which of the following would an accused be automatically tried by jury?a) Disturbing the peaceb) Murder chargesc) Drug possession chargesd) Assault and battery chargese)Trespassing charges why the internet is not policed How many moles of oxygen are required to react completely with 5 mol C8H18? working alone, aliyah can dig a 10ft by 10 ft hole is 6 hours. one day her friend eugene helped her and it only took 3.75 hours. how long would it take eugene to do it alone? disclaimer: the answer is 11.95 hours, but i keep getting 10. please use the 1/x+1/y=1/z strategy please help, will give brainliest!!! someone help me pls i need to pass summer school given m||n, find the value of x Algebra II Part 1Choose the expression or equation that correctly represents this informationRose works eight hours a day for five days a week. How many hours will she work in saweeks?hours = 40 = 6hours = 40.6hours = 6 = 40 Write down at least five number pairs to solve the equation vw = 50